Diễn Đàn MathScopeDiễn Đàn MathScope
  Diễn Đàn MathScope
Ghi Danh Hỏi/Ðáp Community Lịch

Go Back   Diễn Đàn MathScope > Sơ Cấp > Tài Liệu > Đề Thi > Đề Thi HSG Cấp Tỉnh ở Việt Nam

News & Announcements

Ngoài một số quy định đã được nêu trong phần Quy định của Ghi Danh , mọi người tranh thủ bỏ ra 5 phút để đọc thêm một số Quy định sau để khỏi bị treo nick ở MathScope nhé !

* Nội quy MathScope.Org

* Một số quy định chung !

* Quy định về việc viết bài trong diễn đàn MathScope

* Nếu bạn muốn gia nhập đội ngũ BQT thì vui lòng tham gia tại đây

* Những câu hỏi thường gặp

* Về việc viết bài trong Box Đại học và Sau đại học


Trả lời Gởi Ðề Tài Mới
 
Ðiều Chỉnh Xếp Bài
Old 08-11-2012, 07:26 PM   #1
hoangquan_9x
+Thành Viên+
 
hoangquan_9x's Avatar
 
Tham gia ngày: Nov 2011
Đến từ: THPT Hồng Thái_Hà Nội
Bài gởi: 171
Thanks: 178
Thanked 88 Times in 48 Posts
Đề thi chọn đội tuyển HSG TP Hà Nội (Vòng 2)

Bài I (4điểm)
1, Chứng minh rằng các số $3,3^2,3^3,....,3^{41}$ khi chia cho $83$ được $41$ số dư khác nhau.

2, Cho $a,b,c$ là các hằng số dương .Giải HPT sau:

$$\left\{\begin{matrix}
ax+by=(x-y)^2 & & \\
by+cz=(y-z)^2 & &\\
cz+ax=(z-x)^2 & &
\end{matrix}\right.$$

Bài II (4điểm)
Cho 3 số dương $a,b,c$ thỏa mãn $a+b+c=6$.Tìm GTLN của biểu thức.

$$P=(3-a)(3-b)(3-c)(\dfrac{1}{b^2c^2}+\dfrac{1}{c^2a^2}+\dfrac{1}{a ^2b^2})$$

Bài III (4điểm)
Cho dãy số $x_{1}=20,x_{2}=30,x_{n+2}=3x_{n+1}-x_{n}$, với $n\in N,n \geq 1$.Tìm tất cả các giá trị của $n$ để $5x_{n+1}.x_{n}+1$ là số chính phương.

Bài IV (4điểm)
Cho tứ giác $ABCD$ có các cặp cạnh đối không song song và nội tiếp đường tròn tâm $O$ .Gọi $E,F,I$ tương ứng là giao điểm của các đường thẳng $AB$ và $CD$ , $AD$ và $BC$, $AC$ và $BD$ .Đường phân giác của các góc $\widehat{AED} $ và $ \widehat{AFB}$ cắt nhau tại $H$ .Gọi $K$ là điểm chung thứ hai của đường tròn ngoại tiếp tam giác $ABI$ và đường tròn ngoại tiếp ta giác $CDI$ .Chứng minh $4$ điểm $E,F,H,K$ nằm trên một đường tròn.

Bài V (4điểm)
Xác định hàm số liên tục $f:R_{+}^{*}\rightarrow R_{+}^{*}$ thỏa mãn đồng thời các điều kiện sau:

1, $f(2x)=2f(x)$

2, $f\left ((f(x))^3(e^{f(x)}-1) \right )=x^2(e^x-1)f(x)$, $\forall x \in R_{+}^{*} $

3, $f(e-1)=(e-1)f(1)$

4, $f(k)$ là số nguyên dương với mọi số nguyên $k$

($R_{+}^{*}$ là tập các số thực dương)


[RIGHT][I][B]Nguồn: MathScope.ORG[/B][/I][/RIGHT]
 
__________________
Không có chữ kí

thay đổi nội dung bởi: hoangquan_9x, 08-11-2012 lúc 07:39 PM
hoangquan_9x is offline   Trả Lời Với Trích Dẫn
The Following 9 Users Say Thank You to hoangquan_9x For This Useful Post:
boykhtna1 (08-11-2012), High high (08-11-2012), hoangthuygiang (04-09-2016), hongson_vip (09-11-2012), huynhcongbang (08-11-2012), Ispectorgadget (08-11-2012), math_lover_hb (09-11-2012), Shuichi Akai (08-11-2012), starandsky1995 (08-11-2012)
Old 08-11-2012, 08:09 PM   #2
tangchauphong
+Thành Viên+
 
tangchauphong's Avatar
 
Tham gia ngày: Jul 2011
Đến từ: MC online
Bài gởi: 159
Thanks: 208
Thanked 62 Times in 52 Posts
Bài I:
1, Do 83 là số nguyên tố. Ta có $3^{82} $
Suy ra $(3^{41}-1)(3^{41}+1) \equiv 0 mod 83 $
Mà ta chứng minh được $(3^{41}+1) $ không chia hết cho 83.
Suy ra $(3^{41}-1) $ chia hết cho 83.
Giả sử các số $3, 3^2, ..., 3^{41} $ chia cho 83 có ít hơn 41 số dư khác nhau.
Khi đó tồn tại 2 số $3^a, 3^b $ ($1 \le a < b \le 41 $) sao cho
$3^b - 3^a $ chia hết cho 83.
Mà $(3;83)=1 $ Suy ra $3^{b-a} -1 $ chia hết cho 83.
Ta dễ dàng chứng minh được 41 là cấp số của 3 mod 83.
Do đó 41 chia hết cho $(b-a) $.
Vậy $b-a=1 $ nhưng $b-a=1 $ không thỏa mãn.
Vậy điều giả sử là sai.
Do đó ta có đpcm.
[RIGHT][I][B]Nguồn: MathScope.ORG[/B][/I][/RIGHT]
 
__________________
tangchauphong is offline   Trả Lời Với Trích Dẫn
Old 08-11-2012, 08:25 PM   #3
boykhtna1
+Thành Viên+
 
boykhtna1's Avatar
 
Tham gia ngày: Feb 2011
Bài gởi: 5
Thanks: 41
Thanked 0 Times in 0 Posts
BÀI 3 xem tại đây: [Only registered and activated users can see links. ]
[RIGHT][I][B]Nguồn: MathScope.ORG[/B][/I][/RIGHT]
 
boykhtna1 is offline   Trả Lời Với Trích Dẫn
Old 08-11-2012, 09:15 PM   #4
hoangquan_9x
+Thành Viên+
 
hoangquan_9x's Avatar
 
Tham gia ngày: Nov 2011
Đến từ: THPT Hồng Thái_Hà Nội
Bài gởi: 171
Thanks: 178
Thanked 88 Times in 48 Posts
Trích:
Nguyên văn bởi hoangquan_9x View Post

Bài II (4điểm)
Cho 3 số dương $a,b,c$ thỏa mãn $a+b+c=6$.Tìm GTLN của biểu thức.
Mình trình bày bài này hơi trâu ( Ai có cách khác trình bày để mọi người học hỏi thêm nhé.
Ta cần chứng minh $\Leftrightarrow (a^2+b^2+c^2)(a+b-c)(b+c-a)(c+a-b)a\leq 3a^2b^2c^2$
Ta đi chứng minh một BĐT mạnh hơn:

$$(a^3+b^3+c^3+abc)(a+b-c)(c+a-b)(b+c-a)\leq 4a^2b^2c^2$$
Không giảm tính tổng quát giả sử $a,b,c$ là độ dài 3 cạnh của tam giác.

Đặt $a+b-c=x.a+c-b=y.b+c-a=z$ ($x,y,z\geq 0$)

Và ta dễ thấy $x,y,z $ cũng là độ dài 3 cạnh của 1 tam giác
(Vì $x-y <z,y-z<x,z-x<y$)

Khi đó.

$BĐT \Leftrightarrow (x+y)^2(y+z)^2(z+x)^2 \geq 4(x^3+y^3+z^3+2(x+y)(y+z)(z+x)-3xyz)xyz$

Chuyển về ngôn ngữ $p,q,r$ với chuẩn hóa $p=x+y+z=3$ Ta được

$BĐT \Leftrightarrow 3q^2+2qr+3r^2-36r\geq 0$

Đặt $f(r)=VT=3r^2+2r(q-18)+3q^2$ có $f'(r)=6r+2q-36<0$

Suy ra $f(r)$ là hàm nghịch biến.

* Mặt khác với $x,y,z$ là độ dài 3 cạnh của 1 tam giác.

Áp dụng hệ quả BĐT schur trong tam giác ta được: $r\leq \dfrac{p(5q-p^2)}{18}=\dfrac{5q-9}{6}$

$\Rightarrow f(r)\geq f(\dfrac{5q-9}{6})=\dfrac{27(q-3)^2}{4}\geq 0 $ (dpcm)

Đẳng thức xảy ra khi $a=b=c$

Như vậy ta cần chứng minh

$4(a^2+b^2+c^2)(a+b+c)\leq 9(a^3+b^3+c^3+abc)$

$\Leftrightarrow 5(a^3+b^3+c^3)+9abc\geq 4[ab(a+b)+bc(b+c)+ac(c+a)]$
Nhưng BĐT trên lại chính là BĐT schur trong tam giác
Suy ra $dpcm$

Vậy Max$P$=$\dfrac{3}{16}$ đẳng thức xảy ra khi $a=b=c=2$
[RIGHT][I][B]Nguồn: MathScope.ORG[/B][/I][/RIGHT]
 
__________________
Không có chữ kí
hoangquan_9x is offline   Trả Lời Với Trích Dẫn
The Following 2 Users Say Thank You to hoangquan_9x For This Useful Post:
hamaianh0405 (09-11-2012), huynhcongbang (08-11-2012)
Old 08-11-2012, 09:56 PM   #5
tranghieu95
+Thành Viên+
 
tranghieu95's Avatar
 
Tham gia ngày: Oct 2010
Đến từ: THPT Phan Bội Châu- Nghệ An
Bài gởi: 382
Thanks: 187
Thanked 364 Times in 197 Posts
Gửi tin nhắn qua Yahoo chát tới tranghieu95
1 lời giải khác cho bài bđt ở VMF
[Only registered and activated users can see links. ]
[RIGHT][I][B]Nguồn: MathScope.ORG[/B][/I][/RIGHT]
 
__________________
TỪ TỪ LÀ HẠNH PHÚC
A1K39
XIN LỖI ĐÃ THẤT HỨA NHÉ

KỆ
tranghieu95 is offline   Trả Lời Với Trích Dẫn
Old 08-11-2012, 09:58 PM   #6
huynhcongbang
Administrator

 
huynhcongbang's Avatar
 
Tham gia ngày: Feb 2009
Đến từ: Ho Chi Minh City
Bài gởi: 2,413
Thanks: 2,165
Thanked 4,188 Times in 1,381 Posts
Gửi tin nhắn qua Yahoo chát tới huynhcongbang
Trích:
Nguyên văn bởi hoangquan_9x View Post
2, Cho $a,b,c$ là các hằng số dương .Giải HPT sau:

$$\left\{\begin{matrix}
ax+by=(x-y)^2 & & \\
by+cz=(y-z)^2 & &\\
cz+ax=(z-x)^2 & &
\end{matrix}\right.$$

Bài III (4điểm)
Cho dãy số $x_{1}=20,x_{2}=30,x_{n+2}=3x_{n+1}-x_{n}$, với $n\in N,n \geq 1$.Tìm tất cả các giá trị của $n$ để $5x_{n+1}.x_{n}+1$ là số chính phương.
Bài 2. Bài này trước đây có xuất hiện trong đề chuyên Quang Trung.
Có thể giải bài này như sau:

Ta tính được
$ax = \frac{(x-y)^2+(x-z)^2-(y-z)^2}{2} = (x-y)(x-z)$.
$by = (y-z)(y-x), cz = (z-x)(z-y).$

Suy ra $(ax)(by)(cz) =- (x-y)^2(y-z)^2(z-x)^2 \le 0$. (1)

Rõ ràng tổng của 2 trong 3 số $ax, by, cz$, đều không âm.

Giả sử $ax < 0$ hay $x < 0 $ thì suy ra $y > 0, z > 0$, nhưng khi đó $ax = (x-y)(x-z) > 0$, mâu thuẫn.
Do đó $ax \ge 0.$ Tương tự, ta cũng có $by, cz \ge 0$.
Khi đó $(ax)(by)(cz) \ge 0$. (2)

Từ (1) và (2), ta thấy $(ax)(by)(cz)=0$ hay $xyz=0$.
Ta xét các trường hợp:
- Nếu $x=0$ thì hệ đã cho trở thành
$$\left\{\begin{matrix}
by=y^2 & & \\
by+cz=(y-z)^2 & &\\
cz=z^2 & &
\end{matrix}\right.$$

Từ hệ này suy ra $y^2+z^2=(y-z)^2$ hay $yz=0$.
+Nếu $y=0$ thì $cz=z^2$ nên $z=0$ hoặc $z=c$.
Hệ có nghiệm $(x,y,z)=(0,0,0), (0,0,z)$.
Tương tự với các trường hợp khác, ta có được 4 nghiệm của hệ đã cho là $(x,y,z)=(0,0,0), (a,0,0), (0,b,0),(0,0,c).$

Bài III.
Bài này tìm công thức tổng quát của dãy số ra rồi tìm điều kiện cũng khá dài dòng.
PT đặc trưng là $t^2=3t-1$ có nghiệm là $t = \frac{3 \pm \sqrt{5}}{2}.$ (*)
Ta tìm được CTTQ là $x_n = 10(t_1^{n-1}+t_2^{n-1})$ với $t_1, t_2$ là nghiệm của PT (*).
Suy ra
$A = 5x_n x_{n+1} +1= 500(x_1^n+x_2^n)(x_1^{n-1}+x_2^{n-1})+1 = 500(x_1^{2n-1}+x_2^{2n-1}+3)+1$.
Chú ý rằng $x_1=y_1^2, x_2=y^2$ với $y_1,y_2$ là nghiệm của phương trình $x^2=x+1$ nên có thể viết lại là $A=500(y_1^{4n-2}+y_2^{4n-2}-2)+2501 = 500(y_1^{2n-1}-y_2^{2n-1})^2+2501$.

Ta chứng minh được $y_1^{2n-1}-y_2^{2n-1} = u \in \mathbb{Z}$ nên đặt $A=v^2, v \in \mathbb{Z}$ thì ta có phương trình $500u^2+2501=v^2$.
[RIGHT][I][B]Nguồn: MathScope.ORG[/B][/I][/RIGHT]
 
__________________
Sự im lặng của bầy mèo

thay đổi nội dung bởi: huynhcongbang, 08-11-2012 lúc 10:38 PM
huynhcongbang is offline   Trả Lời Với Trích Dẫn
The Following 3 Users Say Thank You to huynhcongbang For This Useful Post:
hamaianh0405 (09-11-2012), hoangquan_9x (09-11-2012), nghiepdu-socap (09-11-2012)
Old 09-11-2012, 10:29 AM   #7
hamaianh0405
+Thành Viên+
 
Tham gia ngày: May 2012
Bài gởi: 107
Thanks: 59
Thanked 7 Times in 6 Posts
Trích:
Nguyên văn bởi hoangquan_9x View Post
Mình trình bày bài này hơi trâu ( Ai có cách khác trình bày để mọi người học hỏi thêm nhé.

Ta đi chứng minh một BĐT mạnh hơn:

$$(a^3+b^3+c^3+abc)(a+b-c)(c+a-b)(b+c-a)\leq 4a^2b^2c^2$$

Sao bạn nghĩ được cái này nhỉ mình nghĩ mãi mà không làm được?
[RIGHT][I][B]Nguồn: MathScope.ORG[/B][/I][/RIGHT]
 
hamaianh0405 is offline   Trả Lời Với Trích Dẫn
Old 09-11-2012, 10:59 PM   #8
hamaianh0405
+Thành Viên+
 
Tham gia ngày: May 2012
Bài gởi: 107
Thanks: 59
Thanked 7 Times in 6 Posts
Trích:
Nguyên văn bởi inbeohong View Post
cái này dùng sờ chu hay sao ý
Ý mình hỏi là ý tường chứ không phải lời giải
[RIGHT][I][B]Nguồn: MathScope.ORG[/B][/I][/RIGHT]
 
hamaianh0405 is offline   Trả Lời Với Trích Dẫn
Old 09-11-2012, 11:09 PM   #9
inbeohong
+Thành Viên+
 
Tham gia ngày: Nov 2012
Bài gởi: 2
Thanks: 1
Thanked 0 Times in 0 Posts
à. có lẽ mò. thử a=b=c = 2 thay vào thì đc mã là 3/16. xong bạn í làm xuôi. ra cái bước cần cm cái bđt này thì bạn í lại trình bày ngclại cho khoa học thôi. mình nghĩ thế
[RIGHT][I][B]Nguồn: MathScope.ORG[/B][/I][/RIGHT]
 
inbeohong is offline   Trả Lời Với Trích Dẫn
Old 10-11-2012, 07:48 AM   #10
chuongminh
+Thành Viên+
 
Tham gia ngày: Nov 2012
Đến từ: Thành phố Cao Lãnh
Bài gởi: 21
Thanks: 20
Thanked 7 Times in 6 Posts
Mình giải như thế này nhé, các bạn góp ý cho lời giải của mình!
Xét phương trình đặc trưng $${x^2} - 3x + 1$ $ có hai nghiệm là:
$${x_1} = \frac{{3 + \sqrt 5 }}{2}$ $,$${x_2} = \frac{{3 - \sqrt 5 }}{2}$ $
Đặt $$\alpha = \frac{{1 + \sqrt 5 }}{2}\beta = \frac{{1 - \sqrt 5 }}{2}$ $,
Khi đó:
$${\alpha ^2} = {x_1},{\beta ^2} = {x_2}$ $
Với $$\alpha ,\beta $ $ là hai nghiệm của phương trình: $${t^2} - t - 1 = 0$ $ của dãy Fibonaci:
$${F_1} = {F_2} = 1,{F_{n + 2}} = {F_{n + 1}} + {F_n}$ $
Số hạng tổng quát của dãy Fibonaci là
$${F_n} = \frac{1}{{\sqrt 5 }}\left( {{\alpha ^n} - {\beta ^n}} \right)$ $
$${u_n} = 10\left( {{\alpha ^{2n - 2}} + {\beta ^{2n - 2}}} \right)$ $
Vì $$\alpha ,\beta = 1$ $ nên $$1 + {u_{n + 1}}{u_n} = 501 + {\left( {50{F_{2n - 1}}} \right)^2}$ $
Đặt $$1 + {u_{n + 1}}{u_n} = 501 + {\left( {50{F_{2n - 1}}} \right)^2}$ $, k với $$k$ $ nguyên dương, ta được:
$$501 = \left( {k - 50{F_{2n - 1}}} \right)\left( {k + 50{F_{2n - 1}}} \right)$ $
Từ đó: tìm ra được
$$n=3$ $
Vì $${u_3} = 70,{u_4} = 180,1 + 5{u_4}{u_3} = 63001 = {251^2}$ $
[RIGHT][I][B]Nguồn: MathScope.ORG[/B][/I][/RIGHT]
 

thay đổi nội dung bởi: chuongminh, 10-11-2012 lúc 07:59 AM
chuongminh is offline   Trả Lời Với Trích Dẫn
Old 10-11-2012, 08:31 AM   #11
12121993
+Thành Viên+
 
Tham gia ngày: Feb 2012
Bài gởi: 81
Thanks: 23
Thanked 70 Times in 41 Posts
Bài 3 ko cần sử dụng CTTQ, chỉ cần sử dụng đẳng thức $\ x_{n+1}^2-3x_{n+1}x_{n}+x_{n}^2+500=0 $
Suy ra $(\ x_{n+1}+\ x_{n})^2+501= 5\x_{n+1}x_{n}+1 $
Đặt $m=x_{n+1}+x_{n}, 5\x_{n+1}x_{n}+1=n^2 $ ta có $n^2-m^2=501 $. Tìm được $n=251, m=250 $ nên $\ x_{n+1}+\ x_{n}=250 $ và $\ x_{n+1}\ x_{n}=12600 $.
Suy ra $\ x_{n}=70 $ hay $n = 3 $ (do $\ x_{n+1}>\ x_{n} $).
[RIGHT][I][B]Nguồn: MathScope.ORG[/B][/I][/RIGHT]
 

thay đổi nội dung bởi: 12121993, 10-11-2012 lúc 08:38 AM
12121993 is offline   Trả Lời Với Trích Dẫn
The Following 2 Users Say Thank You to 12121993 For This Useful Post:
chuongminh (10-11-2012), hoangkhtn2010 (12-11-2012)
Old 10-11-2012, 11:20 AM   #12
thanhorg
+Thành Viên+
 
thanhorg's Avatar
 
Tham gia ngày: Oct 2011
Đến từ: T1K20- Chuyên Hà Tĩnh
Bài gởi: 213
Thanks: 155
Thanked 145 Times in 89 Posts
Trích:
Nguyên văn bởi hoangquan_9x View Post

Bài V (4điểm)
Xác định hàm số liên tục $f:R_{+}^{*}\rightarrow R_{+}^{*}$ thỏa mãn đồng thời các điều kiện sau:

1, $f(2x)=2f(x)$

2, $f\left ((f(x))^3(e^{f(x)}-1) \right )=x^2(e^x-1)f(x)$, $\forall x \in R_{+}^{*} $

3, $f(e-1)=(e-1)f(1)$

4, $f(k)$ là số nguyên dương với mọi số nguyên $k$

($R_{+}^{*}$ là tập các số thực dương)

Bài 5 : Mình làm thế này :

*CM f đơn ánh :
Thật vậy,giả sử tồn tại a,b sao cho $f(a) = f(b) $
Từ 2 lần lượt thay $x=a,x=b $ ta được $a^2.(e^a-1) =b^2.(e^b-1) $
Suy ra $a=b $ (do $a,b >0 $ và hàm $g(x) = x^2.(e^x-1) $ đồng biến trên $\mathbb{R^+} $ )

Kết hợp f liên tục ta có f là hàm đơn điệu,Mà từ giả thiết 1 ta có $ f(2x) > f(x) $ nên f tăng thực sự

*Tính f(1)
Từ (2) cho x=1 và đặt $f(1) = a $ ta có $f\left(a^3.(e^a - 1) \right) = (e-1).a $
Kết hợp với giả thiết 3 ta được : $f\left(a^3.(e^a - 1) \right) = f(e-1) $
Do f đơn ánh nên $a^3.(e^a-1) = e-1 $
Suy ra a=1 hay $f(1)=1 $

* CM $f(n) = n \ \forall n \in \mathbb{N^*} $ (A)
Thật vậy $f(1) =1 $ nên $f(2) =2,\ f(4) =4 $
nên theo giả thiết 4 và f tăng thực sự ta có $f(3)= 3 $
Ta CM (A) bằng quy nạp
+)n=1,2,3,4 đúng

+)Giả sử đúng tới n = k $(k \ge 4) $
xét khi n = k+1
+) nếu k chẵn tức là k =2m thì $f(k+2) = f(2m+2) = 2f(m+1)=2m+2=k+2 $
nên $f(k+1) =k+1 $ (do giả thiết 4,f tăng thực sự và $f(k) = k ,\\ f(k+2) = k+2 $)
+) nếu k lẻ suy ra $f(k+1) = 2f(\frac{k+1}{2}}) = k+1 $
Như vậy n=k+1 đúng
Theo nguyên lý quy nạp ta có $f(n) = n \ \forall n \in \mathbb{N^*} $

*) CM $f(x) = x \ \forall x \in \mathbb{R_{*}^+} $
Thật vậy Từ giả thiết (1) bằng quy nạp ta có $f(\frac{x}{2^n}) = \frac{1}{2^n}f(x) $
Nên $f(\frac{m}{2^n}) = \frac{m}{2^n} \ \forall m,n \in \mathbb{N^*} $
Do Tập hợp $A = { \frac{m}{2^n} \mid m,n \in \mathbb{N^*} } $ trù mật trong $\mathbb{R^+} $
nên với mọi x thuộc $\mathbb{R^+} $ tồn tại dãy hữu tỷ $(x_n) $ có dạng $\frac{m}{2^n} $ hội tụ về x.
Kết hợp tính liên tục của $f $ ta có ngay $f(x) = x \ \forall x \in \mathbb{R_{*}^+} $
Thử lại thấy hàm số thỏa mãn.

P/s Mọi người check giùm mình cái nhé !!!
[RIGHT][I][B]Nguồn: MathScope.ORG[/B][/I][/RIGHT]
 

thay đổi nội dung bởi: thanhorg, 10-11-2012 lúc 11:25 AM
thanhorg is offline   Trả Lời Với Trích Dẫn
The Following 2 Users Say Thank You to thanhorg For This Useful Post:
einstein1996 (10-11-2012), nghiepdu-socap (10-11-2012)
Old 10-11-2012, 12:30 PM   #13
minhcanh2095
+Thành Viên+
 
minhcanh2095's Avatar
 
Tham gia ngày: Jan 2011
Đến từ: Trường ĐH CNTT - ĐHQG TPHCM
Bài gởi: 574
Thanks: 437
Thanked 256 Times in 159 Posts
Chưa ai chém bài hình hết nhỉ. Gởi mọi người cái hình

Bài này ý tưởng cũng khá rõ ràng. Trước hết chứng minh được $\widehat{FKE}=90$ (dựa vào định lí Brocard). Vậy ta chỉ cần chứng minh $\widehat{FHE}=90$. Mà đây lại là kết quả quen thuôc trong tứ giác nội tiếp.
[RIGHT][I][B]Nguồn: MathScope.ORG[/B][/I][/RIGHT]
 
__________________
Gác kiếm
minhcanh2095 is offline   Trả Lời Với Trích Dẫn
The Following User Says Thank You to minhcanh2095 For This Useful Post:
Raul Chavez (14-11-2012)
Old 10-11-2012, 11:09 PM   #14
tson1997
+Thành Viên+
 
tson1997's Avatar
 
Tham gia ngày: Jun 2012
Đến từ: K46 T1 chuyên SP
Bài gởi: 46
Thanks: 42
Thanked 51 Times in 24 Posts
Cách khác bài II:
Trước hết,ta biến đổi P về dạng:

$8P = \frac{(a+b-c)(b+c-a)(c+a-b)(a^2+b^2+c^2)}{a^2b^2c^2}$

Bây giờ ta sẽ cmr:
$$ (a+b+c)(a+b-c)(b+c-a)(c+a-b)(a^2+b^2+c^2) \le 9a^2b^2c^2 $$ (1)

Thật vậy,nếu $$ (a+b-c)(b+c-a)(c+a-b) \le 0 $$ đương nhiên bđt trên đúng
Nếu $$ (a+b-c)(b+c-a)(c+a-b) > 0 $$ ta thấy rằng 3 số a+b-c;b+c-a;c+a-b không thể có đồng thời 2 số âm.Vì thế,ta có thể quy bài toán về cminh bđt đúng với 3 cạnh 1 tam giác.

Ta xét tam giác ABC với bán kính ngoại tiếp R;a=BC;b=CA;c=AB.Ta có :

$$9R^2 \ge a^2+b^2+c^2$$ (hệ quả của công thức tính OG^2 )

hay $$\frac{9a^2b^2c^2}{16S^2} \ge (a^2+b^2+c^2)$$
hay $$9a^2b^2c^2 \ge (a+b+c)(a+b-c)(b+c-a)(c+a-b)(a^2+b^2+c^2)$$

Vậy ta có (1) đc cminh.
Suy ra $$8P \le \frac{9}{6} =\frac{3}{2} $$
hay $$ P \le \frac{3}{16} $$
[RIGHT][I][B]Nguồn: MathScope.ORG[/B][/I][/RIGHT]
 
tson1997 is offline   Trả Lời Với Trích Dẫn
The Following User Says Thank You to tson1997 For This Useful Post:
hosyhaiql (08-12-2012)
Old 30-08-2014, 07:28 PM   #15
Nvthe_cht.
+Thành Viên+
 
Tham gia ngày: Nov 2013
Bài gởi: 69
Thanks: 15
Thanked 36 Times in 24 Posts
Câu 3. Có thể sử dụng hằng đẳng thức:
$U_{n+1}.U_{n-1}-U_n^2=U_3.U_1-U_2^2$
[RIGHT][I][B]Nguồn: MathScope.ORG[/B][/I][/RIGHT]
 
Nvthe_cht. is offline   Trả Lời Với Trích Dẫn
The Following User Says Thank You to Nvthe_cht. For This Useful Post:
thaygiaocht (30-08-2014)
Trả lời Gởi Ðề Tài Mới

Bookmarks


Quuyền Hạn Của Bạn
You may not post new threads
You may not post replies
You may not post attachments
You may not edit your posts

BB code is Mở
Smilies đang Mở
[IMG] đang Mở
HTML đang Tắt

Chuyển đến


Múi giờ GMT. Hiện tại là 08:23 PM.


Powered by: vBulletin Copyright ©2000-2024, Jelsoft Enterprises Ltd.
Inactive Reminders By mathscope.org
[page compression: 112.60 k/128.96 k (12.69%)]